LSAT and Law School Admissions Forum

Get expert LSAT preparation and law school admissions advice from PowerScore Test Preparation.

User avatar
 Dave Killoran
PowerScore Staff
  • PowerScore Staff
  • Posts: 5972
  • Joined: Mar 25, 2011
|
#94165
Complete Question Explanation
(The complete setup for this game can be found here: viewtopic.php?f=393&t=1212)

The correct answer choice is (E)

The question stem establishes the following information:

G4-Q24-d1.png

Because O and S participate in both the first and third years, from the second rule they cannot participate in the second year. With O and S removed from consideration, N, P, and T must be the three clans that participate in the second year:

G4-Q24-d2.png

With T participating in both the second and third years, according to the second rule T cannot participate in the fourth year, and thus to meet the conditions of the first rule T must participate in the fifth year:

G4-Q24-d3.png

Only answer choice (E) contains T among its list of clans participating in the fifth year, and thus answer choice (E) must be correct.

Note that we can also infer that P participates in the fourth and fifth years, but that alone does not prove any individual answer choice correct, although it does eliminate answer choice (B).
You do not have the required permissions to view the files attached to this post.
 AArbo031
  • Posts: 3
  • Joined: Dec 13, 2020
|
#98538
Hi,

I've been trying to work out this problem for a while, but I cannot figure out how the correct answer is E. I've uploaded an attachment where I added my work onto your explanation, but my answer isn't even listed amongst the answer choices. :-?

I understand why P and T need to go in year 5, but I cannot understand how/why S should be there.
I assumed S needed to be in the 4th year in order for it to meet the first condition of two consecutive years (with S already being in the 3rd year). If S were to not be in the 4th year then it would fail to meet that condition. In addition, if it's placed in the 5th year (along with the 4th year) then it'll appear more than 3 times and will be listed in 3 consecutive years which violates the second and third conditions.

Any help as to what on earth I'm missing is appreciated. :-D
You do not have the required permissions to view the files attached to this post.
 Adam Tyson
PowerScore Staff
  • PowerScore Staff
  • Posts: 5387
  • Joined: Apr 14, 2011
|
#98539
I think you may be misinterpreting the rule about consecutive years, AArbo031 . That rule does not require that S, or any other particular clan, appear in consecutive years. It means that any clan that does not appear in one year must appear in the next year. Every clan appears at least once in any two consecutive years.

For that reason, your hypothetical solution fails because N does not appear in either year 3 or year 4. You would have to put N in year 4 to be sure that it appears at least once in any two consecutive years!

As far as S goes, it does not have to appear in year 5. It could instead be in year 4 with N and P, making year 5 be T, O, and P. But this wasn't a Must Be True question - it was a COULD be true question! S COULD appear in year 5. But it's a guarantee that T and P are both in year 5, and answer E is the only one that has both. Thus, it is the only answer choice out of the five we were given that could be true, and the others must all be false.

Get the most out of your LSAT Prep Plus subscription.

Analyze and track your performance with our Testing and Analytics Package.